Difference between revisions of "2013 USAJMO Problems/Problem 1"
(→Solution) |
|||
Line 12: | Line 12: | ||
Otherwise, either <math>a^5b\equiv 5\pmod 9</math> or <math>a^5b\equiv 7\pmod 9</math>. Note that since <math>a^6b^6</math> is a perfect sixth power, and since neither <math>a</math> nor <math>b</math> contains a factor of <math>3</math>, <math>a^6b^6\equiv 1\pmod 9</math>. If <math>a^5b\equiv 5\pmod 9</math>, then <cmath>a^6b^6\equiv (a^5b)(ab^5)\equiv 5ab^5\equiv 1\pmod 9\implies ab^5\equiv 2\pmod 9.</cmath> Similarly, if <math>a^5b\equiv 7\pmod 9</math>, then <cmath>a^6b^6\equiv (a^5b)(ab^5)\equiv 7ab^5\equiv 1\pmod 9\implies ab^5\equiv 4\pmod 9.</cmath> Therefore <math>ab^5+3\equiv 5,7\pmod 9</math>, contradiction. | Otherwise, either <math>a^5b\equiv 5\pmod 9</math> or <math>a^5b\equiv 7\pmod 9</math>. Note that since <math>a^6b^6</math> is a perfect sixth power, and since neither <math>a</math> nor <math>b</math> contains a factor of <math>3</math>, <math>a^6b^6\equiv 1\pmod 9</math>. If <math>a^5b\equiv 5\pmod 9</math>, then <cmath>a^6b^6\equiv (a^5b)(ab^5)\equiv 5ab^5\equiv 1\pmod 9\implies ab^5\equiv 2\pmod 9.</cmath> Similarly, if <math>a^5b\equiv 7\pmod 9</math>, then <cmath>a^6b^6\equiv (a^5b)(ab^5)\equiv 7ab^5\equiv 1\pmod 9\implies ab^5\equiv 4\pmod 9.</cmath> Therefore <math>ab^5+3\equiv 5,7\pmod 9</math>, contradiction. | ||
+ | |||
+ | Therefore no such integers exist. | ||
+ | |||
+ | ==Solution 2== | ||
+ | We shall prove that such integers do not exist via contradiction. | ||
+ | Suppose that <math>a^5b + 3 = x^3</math> and <math>ab^5 + 3 = y^3</math> for integers a and b. Rearranging terms gives <math>a^5b = x^3 - 3</math> and <math>ab^5 = y^3 - 3</math>. Solving for a and b (by first multiplying the equations together and taking the sixth root) gives a = <math>(x^3 - 3)^\frac{5}{24} (y^3 - 3)^\frac{-1}{24}</math> and b = <math>(y^3 - 3)^\frac{5}{24} (x^3 - 3)^\frac{-1}{24}</math>. Consider a prime p in the prime factorization of <math>x^3 - 3</math> and <math>y^3 - 3</math>. If it has power r_1 in <math>x^3 - 3</math> and power r_2 in <math>y^3 - 3</math>, then 5r_1 - r_2 divides 24 and 5r_2 - r_1 also divides 24. Adding and subtracting the divisions gives that r_1 - r_2 divides 12. Because 5r_1 - r_2 also divides 12, 4r_1 divides 12 and thus r_1 divides 3. Repeating this trick for all primes in <math>x^3 - 3</math>, we see that <math>x^3 - 3</math> is a perfect cube, say <math>q^3</math>. Then <math>x^3 - q^3 = 3,</math> and $(x-q)(x^2 + xq + q^2) = 3, so that x - q = 1 and x^2 + xq + q^2 = 3. Clearly, this system of equations has no integer solutions for x or q, a contradiction, hence completing the proof. | ||
Therefore no such integers exist. | Therefore no such integers exist. | ||
{{MAA Notice}} | {{MAA Notice}} |
Revision as of 12:40, 16 March 2014
Problem
Are there integers and such that and are both perfect cubes of integers?
Solution
No, such integers do not exist. This shall be proven by contradiction, by showing that if is a perfect cube then cannot be.
Remark that perfect cubes are always congruent to , , or modulo . Therefore, if , then .
If , then note that . (This is because if then .) Therefore and , contradiction.
Otherwise, either or . Note that since is a perfect sixth power, and since neither nor contains a factor of , . If , then Similarly, if , then Therefore , contradiction.
Therefore no such integers exist.
Solution 2
We shall prove that such integers do not exist via contradiction. Suppose that and for integers a and b. Rearranging terms gives and . Solving for a and b (by first multiplying the equations together and taking the sixth root) gives a = and b = . Consider a prime p in the prime factorization of and . If it has power r_1 in and power r_2 in , then 5r_1 - r_2 divides 24 and 5r_2 - r_1 also divides 24. Adding and subtracting the divisions gives that r_1 - r_2 divides 12. Because 5r_1 - r_2 also divides 12, 4r_1 divides 12 and thus r_1 divides 3. Repeating this trick for all primes in , we see that is a perfect cube, say . Then and $(x-q)(x^2 + xq + q^2) = 3, so that x - q = 1 and x^2 + xq + q^2 = 3. Clearly, this system of equations has no integer solutions for x or q, a contradiction, hence completing the proof.
Therefore no such integers exist. The problems on this page are copyrighted by the Mathematical Association of America's American Mathematics Competitions.